$271$

Il forum di Base5, dove è possibile postare problemi, quiz, indovinelli, rompicapo, enigmi e quant'altro riguardi la matematica ricreativa e oltre.

Moderatori: Gianfranco, Bruno

panurgo
Livello 9
Livello 9
Messaggi: 1521
Iscritto il: sab nov 19, 2005 3:45 pm
Località: Padova

$271$

Messaggio da panurgo »

Esprimere $271$ come somma di numeri reali positivi in modo da massimizzare il prodotto di tali numeri.
il panurgo

Principio di Relatività: $\mathbb{m} \not \to \mathbb{M} \, \Longleftrightarrow \, \mathbb{M} \not \to \mathbb{m}$
"Se la montagna non va a Maometto, Maometto NON va alla montagna"

delfo52
Livello 9
Livello 9
Messaggi: 1556
Iscritto il: mer mag 25, 2005 4:19 pm
Località: bologna

Re: $271$

Messaggio da delfo52 »

dopo aver costatato a occhio che 90x90x90 è più piccolo di 60x60x60x60 , a sua volta più piccolo di 50x50x50x50x50, sparo senza fare ulteriori calcoli: 2x2x2x2x2x2...x3
Enrico

panurgo
Livello 9
Livello 9
Messaggi: 1521
Iscritto il: sab nov 19, 2005 3:45 pm
Località: Padova

Re: $271$

Messaggio da panurgo »

Sei circa allo $0,33\ \%$ del massimo... :wink:
il panurgo

Principio di Relatività: $\mathbb{m} \not \to \mathbb{M} \, \Longleftrightarrow \, \mathbb{M} \not \to \mathbb{m}$
"Se la montagna non va a Maometto, Maometto NON va alla montagna"

Info
Livello 5
Livello 5
Messaggi: 377
Iscritto il: lun nov 21, 2005 1:11 pm
Contatta:

Re: $271$

Messaggio da Info »

io farei 1.5×1.5×1.5......(180 volte) per ottenere il massimo

panurgo
Livello 9
Livello 9
Messaggi: 1521
Iscritto il: sab nov 19, 2005 3:45 pm
Località: Padova

Re: $271$

Messaggio da panurgo »

...ma $180\cdot1,5=270$ :shock:
il panurgo

Principio di Relatività: $\mathbb{m} \not \to \mathbb{M} \, \Longleftrightarrow \, \mathbb{M} \not \to \mathbb{m}$
"Se la montagna non va a Maometto, Maometto NON va alla montagna"

Info
Livello 5
Livello 5
Messaggi: 377
Iscritto il: lun nov 21, 2005 1:11 pm
Contatta:

Re: $271$

Messaggio da Info »

beh avrei da moltiplicare per 1 dopo.... non cambia nulla

Info
Livello 5
Livello 5
Messaggi: 377
Iscritto il: lun nov 21, 2005 1:11 pm
Contatta:

Re: $271$

Messaggio da Info »

forse meglio 1.5x179+2.5....

panurgo
Livello 9
Livello 9
Messaggi: 1521
Iscritto il: sab nov 19, 2005 3:45 pm
Località: Padova

Re: $271$

Messaggio da panurgo »

Delfo,

$\displaystyle 2^{134}×3= 65334214448820184984967924626899496599552=6,533\ldots\times 10^{40}$

Info,

$\displaystyle \left(\frac32\right)^{180}\times 1=\frac{76177348045866392339289727720615561750424801402395196724001565744957137343033038019601}{1532495540865888858358347027150309183618739122183602176}=4,970\ldots\times10^{31}$

$\displaystyle \left(\frac32\right)^{179}\times \frac52=\frac{126962246743110653898816212867692602917374669003991994540002609574928562238388396699335}{1532495540865888858358347027150309183618739122183602176}=8,284\ldots\times10^{31}$
il panurgo

Principio di Relatività: $\mathbb{m} \not \to \mathbb{M} \, \Longleftrightarrow \, \mathbb{M} \not \to \mathbb{m}$
"Se la montagna non va a Maometto, Maometto NON va alla montagna"

Info
Livello 5
Livello 5
Messaggi: 377
Iscritto il: lun nov 21, 2005 1:11 pm
Contatta:

Re: $271$

Messaggio da Info »

Panurgo, io ragionavo sul numero di termini.... ho pensato che sicuramente era maggiore di 1 e poi mi è venuto in mente lo 0.5 facendo tutto un ragionamento a mente partendo da 0.333 e da 2, che non saprei neanche riprodurre... è stato un pò un viaggio mentale (((-;

panurgo
Livello 9
Livello 9
Messaggi: 1521
Iscritto il: sab nov 19, 2005 3:45 pm
Località: Padova

Re: $271$

Messaggio da panurgo »

Mi pare che stiate partendo dal presupposto che i numeri reali positivi debbano essere uguali tra loro: non dico che non sia giusto, ma se così è bisogna dimostrarlo...
il panurgo

Principio di Relatività: $\mathbb{m} \not \to \mathbb{M} \, \Longleftrightarrow \, \mathbb{M} \not \to \mathbb{m}$
"Se la montagna non va a Maometto, Maometto NON va alla montagna"

Pasquale
Livello 12
Livello 12
Messaggi: 2853
Iscritto il: mer mag 25, 2005 2:14 am

Re: $271$

Messaggio da Pasquale »

Secondo me, bisogna tradurre il problema in una funzione di cui trovare il valore massimo nell'intervallo a-b.
Questo non mi è riuscito per perdita della memoria degli antichi studi, ma direi comunque che:

alla somma di 100 addendi pari a 2,71 (2,71x100 = 271), corrisponde il massimo prodotto $2,71^{100}$.

Tale soluzione mi è stata suggerita da quel bravo bambino dal nome Decimal Basic:

LET i=1.1
LET mas=1
DO
LET i=i+0.0001
LET x=INT(270/i)
LET y=271-i*x
LET a=y*i^x
IF a>mas THEN
LET mas=a
PRINT i;x;y;a
END IF
loop
END

Di mio c'è la considerazione che i vari addendi, che devono essere utilizzati come dati del prodotto, devono superare il valore dell'unità.
Se si ha voglia, di attendere poco più per il risultato, che comunque non cambia, la quarta riga della routine può essere sostituita con: LET i=i+0.0000001

Autocommento: in realtà, debbo dire che anche io, intuitivamente sono andato alla ricerca di addendi uguali, tutti o anche tutti meno uno: dunque non è detto, o almeno non è dimostrato che tale intuizione dia il risultato giusto. L'unica possibilità di verifica consiste nel fatto che gli addendi tutti uguali, meno uno, forniscono un risultato inferiore rispetto a quelli tutti uguali.
Dunque, mi riservo di ampliare la ricerca su altre possibilità.
Ultima modifica di Pasquale il dom dic 13, 2020 9:21 pm, modificato 2 volte in totale.
_________________

$\text { }$ciao Immagine ciao
E' la somma che fa il totale (Totò)

panurgo
Livello 9
Livello 9
Messaggi: 1521
Iscritto il: sab nov 19, 2005 3:45 pm
Località: Padova

Re: $271$

Messaggio da panurgo »

Per prima cosa dimostriamo che dati $n$ numeri reali positivi a somma costante il loro prodotto è massimo quando tutti i numeri sono uguali.

Se la somma è costante solo $n-1$ numeri possono essere variati indipendentemente: esprimiamo la somma come

$\displaystyle\sum_{i=1}^n{x_i}=S=x_1+x_2+\cdots+x_{n-1}+\left(S-x_1-x_2+\cdots-x_{n-1}\right)$

Il prodotto può essere dunque espresso come

$\displaystyle\prod_{i=1}^n{x_i}=x_1 x_2 \cdots x_{n-1} \left(S-x_1-x_2+\cdots-x_{n-1}\right)$

Derivando rispetto alle $n-1$ variabili e uguagliando le derivate a zero otteniamo un sistema di $n-1$ equazioni

$\left\{\begin{array}{lC}
\displaystyle \frac{\partial}{\partial x_1}\prod_{i=1}^n{x_i}=\frac{ x_1 x_2 \cdots x_{n-1}}{x_1} \left(S-x_1-x_2+\cdots-x_{n-1}\right) - x_1 x_2 \cdots x_{n-1} = 0 \\
\displaystyle \frac{\partial}{\partial x_2}\prod_{i=1}^n{x_i}=\frac{ x_1 x_2 \cdots x_{n-1}}{x_2} \left(S-x_1-x_2+\cdots-x_{n-1}\right) - x_1 x_2 \cdots x_{n-1} = 0 \\
\displaystyle \vdots \\
\displaystyle \frac{\partial}{\partial x_{n-1}}\prod_{i=1}^n{x_i}=\frac{ x_1 x_2 \cdots x_{n-1}}{x_{n-1}} \left(S-x_1-x_2+\cdots-x_{n-1}\right) - x_1 x_2 \cdots x_{n-1} = 0
\end{array}\right.$

ovvero, semplificando il semplificabile

$
\left\{\begin{array}{lC}
\displaystyle \left(S-x_1-x_2+\cdots-x_{n-1}\right)-x_1 = 0 \\
\displaystyle \left(S-x_1-x_2+\cdots-x_{n-1}\right)-x_2 = 0 \\
\displaystyle \vdots \\
\displaystyle \left(S-x_1-x_2+\cdots-x_{n-1}\right)-x_{n-1} = 0
\end{array}\right.
\qquad\Longrightarrow\qquad
\left\{\begin{array}{lC}
\displaystyle 2x_1+x_2+\cdots+x_{n-1} = S \\
\displaystyle x_1+2x_2+\cdots+x_{n-1} = S \\
\displaystyle \vdots \\
\displaystyle x_1+x_2+\cdots+2x_{n-1} = S
\end{array}\right.
$

Sommiamo tutte le equazioni membro a membro

$\displaystyle nx_1+nx_2+\cdots+nx_{n-1}=\left(n-1\right)S$

raccogliamo la $n$

$\displaystyle n\left(x_1+x_2+\cdots+x_{n-1}\right)=n\left(S-x_n\right)=\left(n-1\right)S$

e otteniamo

$\displaystyle x_n=\frac{S}{n} $

Consideriamo ora la somma degli altri numeri: anch’essa è costante e vale

$\displaystyle\sum_{i=1}^{n-1}x_i=S^\prime=S-\frac{S}{n}=\frac{n-1}{n}S$

Ripetiamo, mutatis mutandis, il ragionamento: somma

$\displaystyle\sum_{i=1}^{n-1}{x_i}=x_1+x_2+\cdots+x_{n-2}+\left(S^\prime-x_1-x_2+\cdots-x_{n-2}\right)$

prodotto

$\displaystyle\prod_{i=1}^{n-1}{x_i}=x_1 x_2 \cdots x_{n-2} \left(S^\prime-x_1-x_2+\cdots-x_{n-2}\right)$

derivate

$\left\{\begin{array}{lC}
\displaystyle \frac{\partial}{\partial x_1}\prod_{i=1}^{n-1}{x_i}=\frac{ x_1 x_2 \cdots x_{n-2}}{x_1} \left(S^\prime-x_1-x_2+\cdots-x_{n-2}\right) - x_1 x_2 \cdots x_{n-2} = 0 \\
\displaystyle \frac{\partial}{\partial x_2}\prod_{i=1}^{n-1}{x_i}=\frac{ x_1 x_2 \cdots x_{n-2}}{x_2} \left(S^\prime-x_1-x_2+\cdots-x_{n-2}\right) - x_1 x_2 \cdots x_{n-2} = 0 \\
\displaystyle \vdots \\
\displaystyle \frac{\partial}{\partial x_{n-2}}\prod_{i=1}^{n-1}{x_i}=\frac{ x_1 x_2 \cdots x_{n-2}}{x_{n-2}} \left(S^\prime-x_1-x_2+\cdots-x_{n-2}\right) - x_1 x_2 \cdots x_{n-2} = 0
\end{array}\right.$

semplificazione

$
\left\{\begin{array}{lC}
\displaystyle \left(S^\prime-x_1-x_2+\cdots-x_{n-2}\right)-x_1 = 0 \\
\displaystyle \left(S^\prime-x_1-x_2+\cdots-x_{n-2}\right)-x_2 = 0 \\
\displaystyle \vdots \\
\displaystyle \left(S^\prime-x_1-x_2+\cdots-x_{n-2}\right)-x_{n-2} = 0
\end{array}\right.
\qquad\Longrightarrow\qquad
\left\{\begin{array}{lC}
\displaystyle 2x_1+x_2+\cdots-x_{n-2} = S^\prime \\
\displaystyle x_1+2x_2+\cdots-x_{n-2} = S^\prime \\
\displaystyle \vdots \\
\displaystyle x_1+x_2+\cdots-2x_{n-2} = S^\prime
\end{array}\right.
$

somma delle equazioni

$\displaystyle \left(n-1\right)x_1+\left(n-1\right)x_2+\cdots+\left(n-1\right)x_{n-2}=\left(n-2\right)S^\prime$

fattorizzazione

$\displaystyle\left(n-1\right)\left(x_1+x_2+\cdots+x_{n-2}\right)=\left(n-1\right)\left(S^\prime-x_{n-1}\right)=\left(n-2\right)S^\prime$

per ottenere

$\displaystyle x_{n-1}=\frac{S^\prime}{n-1} =\frac{S}{n} $

Ripetiamo il processo per tutte le volte necessarie ed otteniamo

$\displaystyle x_i=\frac{S}{n}\qquad\forall i $

ovvero

$\displaystyle \left(\prod_{i=1}^{n}{x_i}\right)_{\text{max|}n}=\left(\frac{S}{n}\right)^n$

Troviamo quale valore di $n$ massimizza il prodotto

$\displaystyle\frac{\partial}{\partial n}\left(\frac{S}{n}\right)^n=\left(\frac{S}{n}\right)^n\left[\log\left(\frac{S}{n}\right)-1\right]=0$

cioè

$\displaystyle\log{n}=\log{S}-\log{e}\qquad\Longrightarrow\qquad n=\frac{271}{e}=99,695\ldots$

Ma $n$ è un intero perciò può essere o $99$ o $100$: testiamo entrambi i valori e otteniamo

$\displaystyle\left(\frac{271}{99}\right)^{99}=1,977\times10^{43}$

e

$\displaystyle\left(\frac{271}{100}\right)^{100}=1,981\times10^{43}$

quindi

$\displaystyle \left(\prod_{i=1}^{n}{x_i}\right)_\text{max}=\left(\frac{271}{100}\right)^{100}\approx2\times10^{43}$
il panurgo

Principio di Relatività: $\mathbb{m} \not \to \mathbb{M} \, \Longleftrightarrow \, \mathbb{M} \not \to \mathbb{m}$
"Se la montagna non va a Maometto, Maometto NON va alla montagna"

delfo52
Livello 9
Livello 9
Messaggi: 1556
Iscritto il: mer mag 25, 2005 4:19 pm
Località: bologna

Re: $271$

Messaggio da delfo52 »

a mia parziale discolpa, vedo che i calcoli giusti sono fuori dalla mia portata. E inoltre, avevo letto male, e consideravo solo numeri interi
:D :D :D :D
Enrico

panurgo
Livello 9
Livello 9
Messaggi: 1521
Iscritto il: sab nov 19, 2005 3:45 pm
Località: Padova

Re: $271$

Messaggio da panurgo »

Rileggendo mi sono accorto che

$
\left\{\begin{array}{lC}
\displaystyle \left(S-x_1-x_2+\cdots-x_{n-1}\right)-x_1 = 0 \\
\displaystyle \left(S-x_1-x_2+\cdots-x_{n-1}\right)-x_2 = 0 \\
\displaystyle \vdots \\
\displaystyle \left(S-x_1-x_2+\cdots-x_{n-1}\right)-x_{n-1} = 0
\end{array}\right.
\qquad\Longrightarrow\qquad
\left\{\begin{array}{lC}
\displaystyle x_1 = \left(S-x_1-x_2+\cdots-x_{n-1}\right) = x_n \\
\displaystyle x_2 = \left(S-x_1-x_2+\cdots-x_{n-1}\right) = x_n \\
\displaystyle \vdots \\
\displaystyle x_{n-1} = \left(S-x_1-x_2+\cdots-x_{n-1}\right) = x_n
\end{array}\right.
$

senza perdere troppo altro tempo... :roll:
il panurgo

Principio di Relatività: $\mathbb{m} \not \to \mathbb{M} \, \Longleftrightarrow \, \mathbb{M} \not \to \mathbb{m}$
"Se la montagna non va a Maometto, Maometto NON va alla montagna"

Pasquale
Livello 12
Livello 12
Messaggi: 2853
Iscritto il: mer mag 25, 2005 2:14 am

Re: $271$

Messaggio da Pasquale »

Carissimo Enrico, a dire il vero, la tua soluzione non era errata da un punto di vista intuitivo e concettuale e mi trovava concorde nelle linee di massima; anzi da quella son partito per approfondire gli esperimenti che poi hanno condotto alla realizzazione della routine che ha prodotto il dato giusto. Naturalmente, la routine cerca e trova, ma non dimostra con certezza e chiarezza la soluzione, che in realtà viene generata lungo un iter che può fornire dei suggerimenti o lungo il quale possono intravvedersi, volendo, alcuni passaggi concettuali (vedi gli inconsapevoli 99 e 100, che però qualcosa mi hanno suggerito quando li ho visti e quando più nessun altro dato si è aggiunto a quelli che scorrevano, dopo che l'ultimo addendo ha eguagliato i precedenti). Naturalmente la routine non rappresenta quanto giustamente richiesto dal nostro grandissimo Prof. che ci onora spesso delle sue strabilianti lezioni, tenendo alto il valore di Base5. :shock: :)

Per gli amanti, la suddetta routine, se lanciata con la doppia precisione, fornisce il seguente risultato equivalente nelle approssimazioni:

19812035041219064508530456259880078387903430.23957483215095523344132560728240686527518070815402347619959368389992113799578326163980988950724550534778211063998301353433226397433500342970064583576535924207001101156038726727051691952460732118232001 :mrgreen:

Sempre per gli amanti del Decimal o altro strumento di calcolo, vorrei chiarire perché la routine è stata concepita così come la si vede.
Dovendosi ricercare il massimo valore possibile in riferimento a quanto richiesto dal testo del quesito, appariva evidente che gli addendi con somma 271, dovevano avere un valore superiore all'unità, considerato che successivamente il prodotto avrebbe dovuto rappresentare il massimo possibile.
La ricerca è caduta su addendi fra loro uguali, in modo da poter trasformare il prodotto in una potenza, moltiplicata al massimo per un solo fattore, se non si fosse riusciti a trovare fattori tutti uguali. Dunque, ricerca di un equilibrio fra quantità degli addendi e valore di ciascuno: la maggiore quantità influisce sull'indice della potenza, che come si sa conta molto sulla grandezza di un risultato, ma anche la base ha la sua importanza; infatti le potenze dell'unità, quali siano, producono solo e sempre l'unità. Una base grande ha la sua importanza, ma nel caso specifico, basi maggiori diminuiscono il valore dell'indice. Dunque, ricerca di addendi di valore tale che la loro quantità contenuta nel 271 rappresenti un indice di potenza che con quel valore di base produca il massimo possibile.
In definitiva, la routine esegue la suddetta ricerca velocemente ed in pochi passaggi.
Ultima modifica di Pasquale il dom dic 13, 2020 5:43 pm, modificato 1 volta in totale.
_________________

$\text { }$ciao Immagine ciao
E' la somma che fa il totale (Totò)

Rispondi